Question

Prove that the set of real numbers of the form e^n,n= 0,=+-1,+-2,... is countable.

Prove that the set of real numbers of the form e^n,n= 0,=+-1,+-2,... is countable.

Homework Answers

Know the answer?
Your Answer:

Post as a guest

Your Name:

What's your source?

Earn Coins

Coins can be redeemed for fabulous gifts.

Not the answer you're looking for?
Ask your own homework help question
Similar Questions
Prove whether or not the set ? is countable. a. ? = [0, 0.001) b. ?...
Prove whether or not the set ? is countable. a. ? = [0, 0.001) b. ? = ℚ x ℚ I do not really understand how to prove S is countable.
1. (a) Let S be a nonempty set of real numbers that is bounded above. Prove...
1. (a) Let S be a nonempty set of real numbers that is bounded above. Prove that if u and v are both least upper bounds of S, then u = v. (b) Let a > 0 be a real number. Define S := {1 − a n : n ∈ N}. Prove that if epsilon > 0, then there is an element x ∈ S such that x > 1−epsilon.
Show that the set of all rational numbers of the form n/5, where n is an...
Show that the set of all rational numbers of the form n/5, where n is an integer, is countable?
Show that the set of all rational numbers of the form n/5, where n is an...
Show that the set of all rational numbers of the form n/5, where n is an integer, is countable?
Problem 3 Countable and Uncountable Sets (a) Show that there are uncountably infinite many real numbers...
Problem 3 Countable and Uncountable Sets (a) Show that there are uncountably infinite many real numbers in the interval (0, 1). (Hint: Prove this by contradiction. Specifically, (i) assume that there are countably infinite real numbers in (0, 1) and denote them as x1, x2, x3, · · · ; (ii) express each real number x1 between 0 and 1 in decimal expansion; (iii) construct a number y whose digits are either 1 or 2. Can you find a way...
Let E⊆R (R: The set of all real numbers) Prove that E is sequentially compact if...
Let E⊆R (R: The set of all real numbers) Prove that E is sequentially compact if and only if E is compact
Prove Corollary 4.22: A set of real numbers E is closed and bounded if and only...
Prove Corollary 4.22: A set of real numbers E is closed and bounded if and only if every infinite subset of E has a point of accumulation that belongs to E. Use Theorem 4.21: [Bolzano-Weierstrass Property] A set of real numbers is closed and bounded if and only if every sequence of points chosen from the set has a subsequence that converges to a point that belongs to E. Must use Theorem 4.21 to prove Corollary 4.22 and there should...
Prove that the set of real numbers has the same cardinality as: (a) The set of...
Prove that the set of real numbers has the same cardinality as: (a) The set of positive real numbers. (b) The set of nonnegative real numbers.
Prove that the set of real numbers has the same cardinality as: (a) The set of...
Prove that the set of real numbers has the same cardinality as: (a) The set of positive real numbers. (b) The set of non-negative real numbers.
Prove: Let S be a bounded set of real numbers and let a > 0. Define...
Prove: Let S be a bounded set of real numbers and let a > 0. Define aS = {as : s ∈ S}. Show that inf(aS) = a*inf(S).
ADVERTISEMENT
Need Online Homework Help?

Get Answers For Free
Most questions answered within 1 hours.

Ask a Question
ADVERTISEMENT